yama_sekander
Thanks Received: 4
Vinny Gambini
Vinny Gambini
 
Posts: 24
Joined: January 16th, 2011
 
 
 

Q23 - Despite improvements in treatment

by yama_sekander Thu Jun 30, 2011 4:16 pm

im confused about the answer to this question. I felt that E would be a sufficient assumption, not necessary. I picked C. can someone explain this? Thanks!
User avatar
 
ManhattanPrepLSAT1
Thanks Received: 1909
Atticus Finch
Atticus Finch
 
Posts: 2851
Joined: October 07th, 2009
 
This post thanked 1 time.
 
 

Re: Q23 - Despite improvements in treatment

by ManhattanPrepLSAT1 Fri Jul 01, 2011 12:05 pm

Here's what I think is going on. I agree that the answer choice seems much stronger than a typical necessary assumption and it does fit a better mold as a sufficient assumption, because instead ruling out any one alternative cause, it rules out all possible alternative causes - which seems too strong...

Here's what I think is happening, and my thinking on this has changed recently after an enlightening discussion I had with another instructor at Manhattan LSAT. Notice the question stem says that the "argument depends" on the assumption. Sometimes the question stem says that the "conclusion depends" on the assumption. If the argument depends on the assumption, then without the assumption, the conclusion does not follow from the evidence, though it still could be true. If the conclusion depends on the assumption, then the without the assumption the conclusion is definitely false.

In this case, in order to conclude that it must instead be that the cause of the increased deaths is the use of bronchial inhalers, we must rule out all possible alternative causes. Otherwise, it's not true, that we must come to that conclusion. So answer choice (E) does represent a necessary assumption of the reasoning underlying the argument.

Lets look at the incorrect answer choices:

(A) would weaken the argument by providing an alternative explanation.
(B) provides one way in which the conclusion would have its impact, but doesn't represent the only way the inhalers could have caused the deaths.
(C) doesn't have to be true in order for the conclusion to be drawn from the evidence. The specific means is not important - instead it's whether the inhalers must represent the cause of the increased deaths.
(D) again provides one of many possible means by which the inhalers would cause the damage, but doesn't need to be the means by which the inhalers actually do lead to death.

So the important thing is that we're relating the evidence to the conclusion, and in order for the conclusion to follow from the evidence (in order for the reasoning to be sound), answer choice (E) does have to be assumed.

Does that make sense? And please do let me know what you think!
 
goriano
Thanks Received: 12
Atticus Finch
Atticus Finch
 
Posts: 113
Joined: December 03rd, 2011
 
 
 

Re: Q23 - despite improvements in treatment for asthma...

by goriano Sun Apr 15, 2012 2:13 pm

mshermn Wrote:Here's what I think is going on. I agree that the answer choice seems much stronger than a typical necessary assumption and it does fit a better mold as a sufficient assumption, because instead ruling out any one alternative cause, it rules out all possible alternative causes - which seems too strong...

Here's what I think is happening, and my thinking on this has changed recently after an enlightening discussion I had with another instructor at Manhattan LSAT. Notice the question stem says that the "argument depends" on the assumption. Sometimes the question stem says that the "conclusion depends" on the assumption. If the argument depends on the assumption, then without the assumption, the conclusion does not follow from the evidence, though it still could be true. If the conclusion depends on the assumption, then the without the assumption the conclusion is definitely false.

In this case, in order to conclude that it must instead be that the cause of the increased deaths is the use of bronchial inhalers, we must rule out all possible alternative causes. Otherwise, it's not true, that we must come to that conclusion. So answer choice (E) does represent a necessary assumption of the reasoning underlying the argument.

Lets look at the incorrect answer choices:

(A) would weaken the argument by providing an alternative explanation.
(B) provides one way in which the conclusion would have its impact, but doesn't represent the only way the inhalers could have caused the deaths.
(C) doesn't have to be true in order for the conclusion to be drawn from the evidence. The specific means is not important - instead it's whether the inhalers must represent the cause of the increased deaths.
(D) again provides one of many possible means by which the inhalers would cause the damage, but doesn't need to be the means by which the inhalers actually do lead to death.

So the important thing is that we're relating the evidence to the conclusion, and in order for the conclusion to follow from the evidence (in order for the reasoning to be sound), answer choice (E) does have to be assumed.

Does that make sense? And please do let me know what you think!


Wouldn't answer choice (A) strengthen the conclusion by ruling out the possibility that urban pollution might be the cause? And so wouldn't it be less extreme than (E) since it's ruling out only one of the possibilities?
 
timmydoeslsat
Thanks Received: 887
Atticus Finch
Atticus Finch
 
Posts: 1136
Joined: June 20th, 2011
 
 
trophy
Most Thanked
trophy
First Responder
 

Re: Q23 - despite improvements in treatment for asthma...

by timmydoeslsat Sun Apr 15, 2012 11:31 pm

The necessary assumption is extreme due to the extreme conclusion: the inhalers must be the cause.

We are given 2 possibilities in the stimulus. And these 2 were shown to not work in giving reasons for the increased rate of deaths from asthma. To conclude that inhalers must be the cause from the evidence given, you need to know that these are the only causes possible.

With answer choice A: We have already established that urban pollution is not the cause. We have shown that when this cause is not present (no pollution) the effect is still there (increased death rates from asthma).

So, we do not need to know that urban pollution has not doubled. It could have increased 100X, it does not matter. We have a situation where no urban pollution is present, yet the effect is present. So this cannot eliminate it as a cause, as it has already been eliminated.
User avatar
 
WaltGrace1983
Thanks Received: 207
Atticus Finch
Atticus Finch
 
Posts: 837
Joined: March 30th, 2013
 
This post thanked 1 time.
 
trophy
Most Thanked
trophy
Most Thankful
trophy
First Responder
 

Re: Q23 - Despite improvements in treatment

by WaltGrace1983 Wed Feb 19, 2014 1:17 pm

I think this question becomes exponentially easier if you understand, as timmy says, the strength of that conclusion. Look at it again:

One must conclude that the cause ... is the use of bronchial inhalers...


But why does this have to be so? This has to be so because the author is assuming something. Namely, the author is assuming that if it is not X and it is not Y then it must be Z. The phrase "must be" is crucial here, absolutely crucial. Without a solid understanding of that then I can see why other answer choices looked better. However (E) gets to what the author is assuming: that there is only 3 possible explanations and, since 1 and 2 are not the case, it must be explanation #3. If we negate the conclusion then the argument looks very fishy:

It is not because of records or urban pollution
+
There are other possible explanations than records, urban pollution, or bronchial inhalers
→
Must be bronchial inhalers

Doesn't look so hot, does it?

As for (C), I would make the argument that it is much too strong and possibly even irrelevant. Maybe the people who are dying due to bronchial inhalers are not using them according to instructions. Thus we would't need to assume (C). In addition, must we assume they are "unsafe?" Maybe they are just unsafe in certain situations or maybe they aren't unsafe at all but people use them inappropriately.
 
seychelles1718
Thanks Received: 0
Atticus Finch
Atticus Finch
 
Posts: 136
Joined: November 01st, 2015
 
 
 

Re: Q23 - Despite improvements in treatment

by seychelles1718 Fri Mar 04, 2016 4:56 am

so when you negate C, it's "inhalers are SAFE." This doesn't impact the conclusion because even if inhalers are safe, it can still CAUSE deaths. Just because something is safe, doesn't mean it CANNOT be the cause of deaths. For example, exercising is safe (or even healthy) but it can still cause deaths for some people.

Is my reasoning correct?

Thanks! :D
User avatar
 
ohthatpatrick
Thanks Received: 3805
Atticus Finch
Atticus Finch
 
Posts: 4661
Joined: April 01st, 2011
 
 
 

Re: Q23 - Despite improvements in treatment

by ohthatpatrick Tue Mar 08, 2016 2:54 pm

Yeah, you're right.

I was sitting here, still unsatisfied with that logic, thinking "Yeah, but if they ARE safe, then how could they be the cause of death?"

But then I read (D) and see how you could still say that something that is intrinsically safe could still be the cause of death (just like your thought --- jogging might cause someone's heart attack, even though jogging is inherently safe).
 
pewals13
Thanks Received: 15
Elle Woods
Elle Woods
 
Posts: 85
Joined: May 25th, 2013
 
 
 

Re: Q23 - Despite improvements in treatment

by pewals13 Thu Dec 01, 2016 2:43 pm

1) What is our task?

To determine what absolutely must be true for the premises provided in the stimulus alone to have any shot at absolutely guaranteeing the argument's conclusion.

2) What is the conclusion?

The cause of increased deaths resultant from asthma is the result of the use of bronchial inhalers by asthma sufferers to relieve their symptoms.

3) What is the support?

The argument provides support for this causal conclusion by eliminating two competing causes.
i. While recording of deaths due to asthma has become more widespread over the same period in which the death rate has has doubled, the rate of death has increased dramatically even in cities with longstanding comprehensive medical records.
ii. Even though there has been an increase in urban pollution over the same period of the rate increase, there has been a dramatic increase in asthma related deaths in cities with longstanding comprehensive medical records and with little or no air pollution.

Essentially, the same result occurred in control groups--meaning the variable is not the cause of the phenomenon.

From a purely logical standpoint--when the LSAT attempts to justify a conclusion by eliminating competing causes, be sure that those are the only possible competing causes.

4) What is the gap?

The argument has not eliminated every potential alternative cause--yet has drawn a causal conclusion.

5) Which answer choices are clearly wrong?

A) "Urban pollution has not doubled in the past decade"
This answer is clearly wrong because this does not need to be assumed--the premises demonstrate that the rate of asthma related deaths still increases at a dramatic rate even in the absence of urban pollution.

B) "Doctors and patients generally ignore the role of allergies in asthma"
This answer choice does not have to be true for the conclusion to hold, even if doctors and patients don't ignore the role of allergies--it's hard to see how this would be relevant to the logic provided in the stimulus regarding a cause of the doubling of the incidence of asthma related deaths.

C) "Bronchial inhalers are unsafe, even when used according to the recommended instructions"
This doesn't necessarily HAVE to be true, maybe bronchial inhalers are invariably used incorrectly and this is the cause of the doubling in the rate of death as a result of asthma. It doesn't have to be true, in a specific sense, that the correct use of bronchial inhalers is unsafe--for the use of bronchial inhalers to be responsible for the increase.

D) "The use of bronchial inhalers aggravates other diseases that frequently occur among asthma sufferers and that often lead to fatal outcomes even when the asthma itself does not"
Like (C), this is not necessary for the conclusion to be proven by the premises given in the stimulus--it is too specific--for the conclusion to hold it simply must be true that the use of bronchial inhalers--for whatever reason, be it aggravation of symptoms or a problem inherent to the inhaler itself when used correctly--is the cause, we don't need more specificity regarding the mechanism by which the inhaler causes the increase in asthma related deaths.

(E) "Increased urban pollution, improved recording of asthma deaths, and the use of bronchial inhalers are the only possible explanations of the increased death rate due to asthma."
For the conclusion to be proven on the basis of the evidence provided, it must be true that the competing causes are the only other alternatives--otherwise the cause in the conclusion cannot be guaranteed, because other causes have not been ruled out.

6) Select the correct answer choice:

E) NEGATION: Increased urban pollution, improved recording of asthma deaths, and the use of bronchial inhalers are not the only possible explanations of increased death rate due to asthma.
If these are not the only possible causal explanations, the conclusion cannot be guaranteed on the basis of the premises provided.
 
echo_rainkey
Thanks Received: 0
Vinny Gambini
Vinny Gambini
 
Posts: 4
Joined: May 07th, 2014
 
 
 

Re: Q23 - Despite improvements in treatment

by echo_rainkey Thu Aug 16, 2018 7:22 pm

I don't understand why D is incorrect.

* The conclusion in the stimulus is "The use of bronchial inhalers causes the increase death."
* Since D is to provide a possible explanation to that "The use of bronchial inhalers causes the increase death", it seems to me a perfect necessary assumption.
* E is too strong( "only") and seems like a sufficient assumption.

Anyone can help fix the problem?
Thanks!